Diferencia entre revisiones de «Compleja:ej-cap1.1»

De luz-wiki
Sin resumen de edición
 
(No se muestran 84 ediciones intermedias de 12 usuarios)
Línea 1: Línea 1:
== 1.1.1 ==
'''1. Demuestre que el producto de números complejos cumple con la ley asociativa'''
'''1. Demuestre que el producto de números complejos cumple con la ley asociativa'''


Línea 27: Línea 25:




--[[Usuario:Gabita|Gabita]] 22:15 28 sep 2009 (UTC)
----
 
Contribución de: [[Usuario:Gabita|Gabita]] 22:15 28 sep 2009 (UTC)
----




== 1.1.2 ==
== '''SECCIÓN 1.1.2''' ==
'''1. Demuestre que <math>\left|\frac{z}{w}\right| = \frac{\left|z\right|}{\left|w\right|}</math>'''
'''1. Demuestre que <math>\left|\frac{z}{w}\right| = \frac{\left|z\right|}{\left|w\right|}</math>'''


Línea 56: Línea 55:
<math>\frac{\left|z\right|}{\left|w\right|} = \frac{\left|a + i b\right|}{\left|c + i d\right|} = \sqrt{\frac{a^2 + b^2}{c^2 + d^2}} = \left|\frac{z}{w}\right|</math>
<math>\frac{\left|z\right|}{\left|w\right|} = \frac{\left|a + i b\right|}{\left|c + i d\right|} = \sqrt{\frac{a^2 + b^2}{c^2 + d^2}} = \left|\frac{z}{w}\right|</math>


--[[Usuario:Gabita|Gabita]] 22:15 28 sep 2009 (UTC)
----
 
Contribución de: [[Usuario:Gabita|Gabita]] 22:15 28 sep 2009 (UTC)


----
----
Línea 89: Línea 88:




--[[Usuario:Josua Da Vinci|Josua Da Vinci]] 23:00 28 sep 2009 (UTC)
----
 
Contribución de:[[Usuario:Josua Da Vinci|Josua Da Vinci]] 23:00 28 sep 2009 (UTC)
 
----
----


Línea 105: Línea 103:




--[[Usuario:Luis Nava|Luis Nava]] 06:35 30 sep 2009 (UTC)
----
Contribución de: [[Usuario:Luis Nava|Luis Nava]] 06:35 30 sep 2009 (UTC)
----


'''5. Sean <math>z_1 , z_2 , z_3 \in \mathbb{C}</math> tales que cumplen $\frac{z_2 - z_1}{z_3 - z_1} = \frac{z_1 - z_3}{z_2 - z_3}$, demuestre que estos tres puntos determinan un triángulo equilátero.'''


----
'''5. Sean <math>z_1 , z_2 , z_3 \in \mathbb{C}</math> tales que cumplen <math>\frac{z_2 - z_1}{z_3 - z_1} = \frac{z_1 - z_3}{z_2 - z_3}</math>, demuestre que estos tres puntos determinan un triángulo equilátero.'''


[[Image: Comp_triang_eq2.jpg|frame|center|Figura 1]]
[[Archivo:Complejaej-cap1.1Triangulo.svg|600px|thumb|center|Figura 1]]


Tenemos que  
Tenemos que  


<center><math>\left | \frac{z_2 - z_1}{z_3 - z_1} \right | = \left | \frac{z_1 - z_3}{z_2 - z_3} \right |,\qquad (1)</math></center>
$<math>\left | \frac{z_2 - z_1}{z_3 - z_1} \right | = \left | \frac{z_1 - z_3}{z_2 - z_3} \right |,\qquad (1)</math>


y, por lo tanto,  
y, por lo tanto,  


<center><math>\frac{|z_2 - z_1|}{|z_3 - z_1|} = \frac{|z_1 - z_3|}{|z_2 - z_3|}.\qquad (2)</math></center>
<math>\frac{|z_2 - z_1|}{|z_3 - z_1|} = \frac{|z_1 - z_3|}{|z_2 - z_3|}.\qquad (2)</math>
   
   
De la Figura 1, vemos que cada una de esas normas de números complejos son exactamente los segmentos de recta que constituyen el triángulo ABC, a saber:
De la Figura 1, vemos que cada una de esas normas de números complejos son exactamente los segmentos de recta que constituyen el triángulo ABC, a saber:


<center><math>\left . \begin{matrix}|z_2 - z_1| = A\\
<center><math>  
|z_3 - z_1| = B = |z_1 - z_3|\\
|z_2 - z_1| = A,\qquad
|z_2 - z_3| = C\\
|z_3 - z_1| = B = |z_1 - z_3|,\qquad
\end{matrix} \right \} \qquad (3)</math></center>
|z_2 - z_3| = C
  \qquad (3)</math></center>


De (2) y (3) tenemos que:
De (2) y (3) tenemos que:
Línea 138: Línea 138:
Y la ecuación (5) es precisamente la condición para que el triángulo ABC de la Figura 1 sea equilátero.
Y la ecuación (5) es precisamente la condición para que el triángulo ABC de la Figura 1 sea equilátero.


--[[Usuario:Belen|Belen]] 02:48 29 sep 2009 (UTC)
----
Contribución de:[[Usuario:Belen|Belen]] 02:48 29 sep 2009 (UTC)


----
----


'''6. Sea <math>{\begin{align}z & = x+iy \end{align}}</math>,  pruebe que
'''6. Sea <math>{z = x+iy }</math>,  pruebe que <math>{\left|{x}\right|+\left|{y}\right|}{\le}{\sqrt{2} \left|{z}\right|}</math>
<math>{\left|{x}\right|+\left|{y}\right|}{\le}{\sqrt{2} \ \left|{z}\right|}</math>




Puesto que el número complejo z puede escribirse como
Puesto que el número complejo z puede escribirse como


<math>{\begin{align}z & = Re(z)+iIm(z) \end{align}}</math>
<math>{z = Re(z)+iIm(z) }</math>


<math>{\begin{align}\left|{z}\right| & = \sqrt{[Re(z)]^2+[Im(z)]^2} \end{align}}</math>
<math>{\left|{z}\right| = \sqrt{[Re(z)]^2+[Im(z)]^2} }</math>




Línea 185: Línea 185:
Como  
Como  


<math>{\begin{align}\left|{z}\right|^2 & = \left|{Re(z)}\right|^2+\left|{Im(z)}\right|^2 \end{align}}</math>
<math>{\left|{z}\right|^2 = \left|{Re(z)}\right|^2+\left|{Im(z)}\right|^2 }</math>


Entonces
Entonces
Línea 204: Línea 204:
<math>{{\sqrt{2} \ \left|{z}\right|}{\ge}{\left|{x}\right|+\left|{y}\right|}}</math>
<math>{{\sqrt{2} \ \left|{z}\right|}{\ge}{\left|{x}\right|+\left|{y}\right|}}</math>


--[[Usuario:Ralf Gutierrez|Ralf Gutierrez]] 19:18 29 sep 2009 (UTC)
----
Contribución de: [[Usuario:Ralf Gutierrez|Ralf Gutierrez]] 19:18 29 sep 2009 (UTC)


----
----


'''6-bis. Sea <math>{\begin{align}z & = x+iy \end{align}}</math>,  pruebe que
'''6-bis. Sea <math>{z = x+iy }</math>,  pruebe que
<math>{\left|{x}\right|+\left|{y}\right|}{\le}{\sqrt{2} \ \left|{z}\right|}</math>
<math>{\left|{x}\right|+\left|{y}\right|}{\le}{\sqrt{2} \ \left|{z}\right|}</math>
   
   
Tenemos que <math>{\begin{align}z & = x+iy \end{align}}</math>, entonces de la teoria sabemos que  
Tenemos que <math>{z = x+iy }</math>, entonces de la teoria sabemos que  




<math>{\begin{align}\left|{z}\right| & = \sqrt{[x]^2+[y]^2} \end{align}}\qquad (1)</math>  
<math>{\left|{z}\right| = \sqrt{[x]^2+[y]^2} }\qquad (1)</math>  


<math>{\left|{x}\right|=\left|{Re(z)}\right|}{\le}{ \left|{z}\right|}</math>
<math>{\left|{x}\right|=\left|{Re(z)}\right|}{\le}{ \left|{z}\right|}</math>
Línea 270: Línea 271:




--[[Usuario:Oscar Adrian|Oscar Adrian]] 03:56 1 oct 2009 (UTC)
----
 
Contribución de: [[Usuario:Oscar Adrian|Oscar Adrian]] 03:56 1 oct 2009 (UTC)
 
----
 
 
 
 
 
 


----
'''REVISADO'''


'''7. Demuestre que en un paralelogramo la suma de los cuadrados de las diagonales es la suma de los cuadrados de los lados.
'''7. Demuestre que en un paralelogramo la suma de los cuadrados de las diagonales es la suma de los cuadrados de los lados.


[[Imagen:Dibujobueno.jpg]]
[[Archivo:Complejaej-cap1.1Paralelogramo.svg|850px|center]]


Sacamos las normas de los números complejos
Sacamos las normas de los números complejos
Línea 325: Línea 320:


<math>|z|^2 +|w|^2 = |h|^2</math>    se cumple la suma de los cuadrados de los lados es igual a la suma de los cuadrados de la diagonal.
<math>|z|^2 +|w|^2 = |h|^2</math>    se cumple la suma de los cuadrados de los lados es igual a la suma de los cuadrados de la diagonal.
--[[Usuario:Karla|Karla]] 22:47 4 oct 2009 (UTC)Sanchez
== 1.1.3 ==
'''1. Calcule las raìces cuadradas de <math>3+4i</math> y de <math>1+2i</math>.'''
Aplicando la formula para calcular raices cuadradas de numeros complejos.
<math>\pm\left(\sqrt{\frac{a+\sqrt{a^2+b^2}}{2}} + i\sqrt{\frac{-a+\sqrt{a^2+b^2}}{2}}\right)</math>  si <math>\quad b>0</math>
Por lo tanto las raices de <math>3+4i</math>, son:
<math>=\pm\left(\sqrt{\frac{3+\sqrt{25}}{2}} + i\sqrt{\frac{-3+\sqrt{25}}{2}}\right)</math>
<math>=\pm\left(\sqrt{\frac{8}{2}} + i\sqrt{1}\right)</math>
<math>=\pm\left(2+i\right)</math>
y para <math>1+2i</math>, son:
<math>=\pm\left(\sqrt{\frac{1+\sqrt{5}}{2}} + i\sqrt{\frac{-1+\sqrt{5}}{2}}\right)</math>
<math>=\pm\left(\sqrt{1.61} + i\sqrt{0.61}\right)</math>
<math>=\pm\left(1.27 + i 0.78\right).</math>
--[[Usuario:Josua Da Vinci|Josua Da Vinci]] 23:44 30 sep 2009 (UTC)
----
'''2.- Calcule las raices sextas de -64 y las raices cubicas de 8i'''
Tenemos q  cosθ+i senθ = -64  definicion en forma polar
[[Imagen:Demo2.jpg]]
r=64
n=6 porque nos piden las raíces sextas
Entonces el argumento θ= ∏
Entonces
Si  x+iy= r exp iφ
Entonces utilizando la definición de Moivre para obtener las raíces
(g  exp iφ)^n= r exp iθ
Ahora tenemos
<math>g^n=r</math>  y    g= raíz enesima <math>\sqrt{64}</math>= = 2
y  φn = θ+2k∏    los 2∏ es porque tomamos en cuenta la periodicidad de la funció n y k son todos los múltiplos de 2∏
entonces sacando las raíces
θ=∏/6  k=0
θ = ∏ / 6 + 2 ∏ / 6 = 3 ∏ / 6    k=1 
θ=∏/6+2(2)∏/6= 5∏/6    k=2 
θ=∏/6+(3)2∏/6= 7∏/6    k=3
θ=∏/6+2(4)∏/6= 9∏/6    k=4
θ=∏/6+2(5)∏/6= 11∏/6    k=1 
Las soluciones son
r1= 2 <math> e^{(i ∏/6)}</math>
r2= 2 <math> e^{(i 3∏/6)}</math>
r3= 2 <math> e^{(i 5∏/6)}</math>
r4= 2 <math> e^{(i 7∏/6)}</math>
r5= 2 <math> e^{(i 9∏/6)}</math>
r6= 2 <math> e^{(i 11∏/6)}</math>
Graficando en coordenadas polares nos queda:
[[Imagen:POLIGONO2.jpg]]
Haciendo algo similar para el 8i Tenemos
cosθ+i senθ= 8i
[[Imagen:DEMO3.jpg]]
el argumento θ= ∏/2
r= 8
n=3 porque nos pinden las raíces cubicas
<math>g^n=r</math>  y    g= raíz enesima <math>\sqrt{8}</math>= = 2
φn = θ+2k∏ 
 
φ=∏/6                k=0
φ=∏/6+2∏/3= 5∏/6    k=1 
φ=∏/6+2(2)∏/3= 9∏/6  k=2 
r1= 2 <math> e^{(i ∏/6)}</math>
r2= 2 <math> e^{(i 5∏/6)}</math>
r3= 2 <math> e^{(i 9∏/6)}</math>
Graficando en coordenadas polares tenemos
[[Imagen:RAICES.jpg]]
--[[Usuario:Karla|Karla]] 21:35 4 oct 2009 (UTC)Sanchez
----
'''2.- Calcule las raices sextas de -64 y las raices cubicas de 8i'''
Sea  <math> z = -64 = 64(cos\pi + isen\pi)\,</math>
Por la formula de De Moivre
<math>z^{1/6} = 64^{1/6}(cos\pi + sen\pi)^{1/6} = 2 (cos(\frac{\pi+2k\pi}{6}) + isen(\frac{\pi+2k\pi}{6}))</math>    para  k = 0,1,2,3,4,5
Evaluando k se obtiene
con k = 0
<math>w_{0} = 2(cos(\frac{\pi}{6}) + isen(\frac{\pi}{6})) = \sqrt{3} + i</math> 
con k = 1
<math>w_{1} = 2(cos(\frac{\pi+2\pi}{6}) + isen(\frac{\pi+2\pi}{6})) = 2(cos(\frac{\pi}{2}) + isen(\frac{\pi}{2})) = 2i</math>
con k = 2 
<math>w_{2} = 2(cos(\frac{\pi+4\pi}{6}) + isen(\frac{\pi+4\pi}{6})) = 2(cos(\frac{5\pi}{6}) + isen(\frac{5\pi}{6})) = -\sqrt{3} + i</math>
con k = 3
<math>w_{3} = 2(cos(\frac{\pi+6\pi}{6}) + isen(\frac{\pi+6\pi}{6})) = 2(cos(\frac{7\pi}{6}) + isen(\frac{7\pi}{6})) = -\sqrt{3} - i</math>
con k = 4
<math>w_{4} = 2(cos(\frac{\pi+8\pi}{6}) + isen(\frac{\pi+8\pi}{6})) = 2(cos(\frac{3\pi}{2}) + isen(\frac{3\pi}{2})) = -2i</math>
con k = 5
<math>w_{5} = 2(cos(\frac{\pi+10\pi}{6}) + isen(\frac{\pi+10\pi}{6})) = 2(cos(\frac{11\pi}{6}) + isen(\frac{11\pi}{6})) = \sqrt{3} - i</math> 
..............
Sea <math>z = 8i = 8(cos(\frac{\pi}{2}) + sen(\frac{\pi}{2})\,</math>
<math>z^{1/3} = 8^{1/3}(cos(\frac{\pi}{2}) + isen(\frac{\pi}{2}))^{1/3} = 2(cos(\frac{\frac{\pi}{2}+2k\pi}{3}) + isen(\frac{\frac{\pi}{2}+2k\pi}{3}))</math>  para k = 0,1,2
Evaluando a k se obtiene
con k = 0
<math>w_{0} = 2(cos(\frac{\pi}{6}) + isen(\frac{\pi}{6}) = \sqrt{3} + i</math>
con k = 1
<math>w_{1} = 2(cos(\frac{\frac{\pi}{2}+2\pi}{3}) + isen(\frac{\frac{\pi}{2}+2\pi}{3})) = 2(cos(\frac{5\pi}{6}) + isen(\frac{5\pi}{6}) = -\sqrt{3} + i</math>
con k = 2
<math>w_{2} = 2(cos(\frac{\frac{\pi}{2}+4\pi}{3}) + isen(\frac{\frac{\pi}{2}+4\pi}{3})) = 2(cos(\frac{3\pi}{2}) + isen(\frac{3\pi}{2})) = -2i</math>
--[[Usuario:Luis Nava|Luis Nava]] 21:07 3 oct 2009 (UTC)
----
'''3. Demuestre que <math>1+Z+Z^2+...+Z^{n-1}=0</math> donde z es una raíz n-ésima de la unidad,
<math>z\neq 1</math>'''
Sea <math> S=1+Z+Z^2+...+Z^{n-1}</math>
Ahora multiplicamos ambos lados por Z
<math> ZS=Z+Z^2+Z^3+...+Z^{n-1}+Z^n</math>
Restando la segunda ecuación de la primera
<math>{(s=1+z+z^2+...+z^{n-1})-(zs=z+^2+z^3+...+z^{n-1}+z^n)}</math>
Tenemos que
<math>{s-zs=1-z^n}\to {s(1-z)=1-z^n}</math>
De donde
<math>s=\frac{1-z^n}{1-z}</math>
Como z es una raíz enesima de la unidad
<math>0=\frac{1-z^n}{1-z}</math>
<math>\to{1-z^n=0} </math>
<math>\to{z^n=1} </math>
Entonces
<math>\to{z^n=1} </math>
y
<math>{1-z}\ne{0}</math>
porque
<math> {z}\ne{1} </math>
Por lo tanto
'''<math> {s=0} </math>'''
--[[Usuario:Ralf Gutierrez|Ralf Gutierrez]] 22:00 2 oct 2009 (UTC)
----
'''4. Demuestre que:
<center>
<math>\frac{n}{2^{n-1}}=\prod_{k=1}^{n-1}\sin\frac{\pi}{k}</math>
</center>
Sugerencia: Factoriza la expresión <math>1+z+z^2+\cdots+z^n</math> usando las raices n-ésimas de la unidad, posteriormente evalue en <math>z=1</math>.'''
Solución:
Las raices de <math>z^m=1</math> son
<center>
<math>z=1,e^{\frac{2\pi i}{m}},e^{\frac{4\pi i}{m}},\dots,e^{\frac{(2m-1)\pi i}{m}}</math>
</center>
entonces podemos escribir
<center>
<math>z^{m-1}=(z-e^{\frac{2\pi i}{m}})(z-e^{\frac{4\pi i}{m}})\cdots(z-e^{\frac{(2m-1)\pi i}{m}})</math>
</center>
dividiendo ambos lados por <math>z-1</math> y haciendo <math>z=1</math>:
<center>
<math>\frac{z^{m-1}}{z-1}=1+z+z^2+\cdots+z^{m-1}</math>
</center>
de aqui hallamos que
<center>
<math>m=(1-e^{\frac{2\pi i}{m}})(1-e^{\frac{4\pi i}{m}})\cdots(1-e^{\frac{(2m-1)\pi i}{m}})\qquad (1)</math>
</center>
tomando el conjugado complejo de ambos lados de (1)
<center>
<math>m=(1-e^{\frac{-2\pi i}{m}})(1-e^{\frac{-4\pi i}{m}})\cdots(1-e^{\frac{-(2m-1)\pi i}{m}})\qquad (2)</math>
</center>
Multiplicando la ecuación (1) por la (2) y aplicando que
<center>
<math>1-(1-e^{\frac{2k\pi i}{m}})(1-e^{\frac{2k\pi i}{m}})= 2 - 2\cos\frac{2k\pi}{m}</math>
</center>
tenemos
<center>
<math>m^2=2^{m-1}(1-\cos\frac{2\pi}{m})(1-\cos\frac{4\pi}{m})\cdots(1-\cos\frac{2(m-1)\pi}{m})</math>
</center>
puesto que
<center>
<math>1-\cos\frac{2k\pi}{m}=2\sin^2\frac{k\pi}{m}</math>
</center>
la ecuación anterior se transforma en
<center>
<math>m^2=2^{2m-2}(\sin^2\frac{\pi}{m})(\sin^2\frac{2\pi}{m})\cdots(\sin^2\frac{(m-1)\pi}{m})</math>
</center>
despejando y sacando la raíz en ambos lados de la expresión:
<center>
<math>\frac{m}{2^{m-1}}=(\sin\frac{\pi}{m})(\sin\frac{2\pi}{m})\cdots(\sin\frac{(m-1)\pi}{m})</math>
</center>
lo que queda demostrada la igualdad.
--[[Usuario:Wendy|Wendy]] 23:10 4 oct 2009 (UTC)
----
'''5.''' '''Demuestre que'''
<math>1+cos\phi+cos2\phi+...............+cos n\phi=\frac{1}{2}+\frac{sen\left(n\phi+\frac{\phi}{2}\right)}{2sen\left(\frac{\phi}{2}\right)}</math>,
'''donde'''          '''<math>\phi</math> '''    '''no es un multiplo par de'''      '''<math>\pi</math>.'''
'''Esta identidad se le atribuye a Lagrange.'''
Sugerencia: calcular la parte real de
<math>1+z+z^{2}+..........+z^{n}</math>,  donde    <math>z=cos\phi+isen\phi</math>.
'''Solucion.'''
Sea
<math>S=1+z+z^{2}+.......+z^{n}</math>            si multiplicamos por <math>z</math> a <math>S</math> se tiene  que
<math>zS=z+z^{2}+z^{3}+......+z^{n}+z^{n+1}</math>          ahora restemos estas dos    ultimas expresiones
<math>\left(S-zS\right)=\left(1-z\right)S=1-z^{n+1}</math>              de lo que se obtiene que
<math>S=\frac{1-z^{n+1}}{1-z}</math>
Si en esta última expresion utilizamos    <math>z=cos\phi+isen\phi</math>    entonces 
<math>S=\frac{1-z^{n+1}}{1-z}</math> 
toma la siguiente forma
<math>1+\left(cos\phi+isen\phi\right)+\left(cos\phi+isen\phi\right)^{2}+........+\left(cos\phi+isen\phi\right)^{n}=\frac{1-\left(cos\phi+isen\phi\right)^{n+1}}{1-\left(cos\phi+isen\phi\right)} </math>
que es equivalente a esta
<math>1+\left(cos\phi+isen\phi\right)+\left(cos2\phi+isen2\phi\right)+........+\left(cosn\phi+isen\left(n\phi\right)\right)=\frac{1-\left(cos\left(n+1\right)\phi+isen\left(n+1\right)\phi\right)}{1-\left(cos\phi+isen\phi\right)}</math>
Tomando el lado derecho de esta ultima expresión y llevar a cabo el producto con su conjugado , es decir:
<math>\left(\frac{1-cos\left(n\phi+\phi\right)-isen\left(n\phi+\phi\right)}{1-cos\phi-isen\phi}\right)\star\left(\frac{1-cos\phi+isen\phi}{1-cos\phi+isen\phi}\right)
</math>
Se obtiene del numerador  lo siguiente
<math>1-cos\phi+isen\phi-cosn\left(n\phi+\phi\right)+cos\phi cos\left(n\phi+\phi\right)-isen\phi cos\left(n\phi+\phi\right)-isen\left(n\phi+\phi\right)+icos\phi sen\left(n\phi+\phi\right)+sen\phi sen\left(n\phi+\phi\right)
</math>
si tomamos solo la parte real se tiene que
<math>1-cos\phi-cos\left(n\phi+\phi\right)+cos\phi cos\left(n\phi+\phi\right)+sen\phi sen\left(n\phi+\phi\right)=</math>
<math>1-cos\phi+cos\left(n\phi-\phi\right)-cos\left(n\phi+\phi\right)</math><math>=</math>  <math>1-cos\phi+2sen\phi sen\phi</math>
por otra parte para el denominador se tiene:
<math>\left(1-cos\phi\right)^{2}+sen^{2}\phi=</math>
<math>1-2cos\phi+sen^{2}\phi+cos^{2}\phi=</math>  <math>2\left(1-cos\phi\right)</math>
al tomar la parte real de
<math>1+\left(cos\phi+isen\phi\right)+\left(cos2\phi+isen2\phi\right)+........+\left(cosn\phi+isenn\phi\right)=\frac{1-\left(cos\left(n+1\right)\phi+isen\left(n+1\right)\phi\right)}{1-\left(cos\phi+isen\phi\right)}</math>,
sustituir lo encontrado para el numerador (parte real)  y el denominador , y utilizar la siguiente
identidad
<math>sen\left(\frac{\phi}{2}\right)=\sqrt{\frac{1-cos\phi}{2}}</math>
tenemos lo siguiente:
<math>
1+cos\phi+cos2\phi+...............+cosn\phi=\frac{2sen\left(\frac{\phi}{2}\right)+sen\phi sen\left(n\phi\right)}{2\left(2sen\left(\frac{\phi}{2}\right)\right)}=\frac{1}{2}+\frac{sen\phi sen\left(n\phi\right)}{2sen\left(\frac{\phi}{2}\right)}</math>
Lo cual es casi a lo que se queria llegar.
--[[Usuario:Dali|Dali]] 00:01 5 oct 2009 (UTC)
== 1.1.4 ==
'''1. Demuestre que:'''
<center><math>
\begin{align}\left | \textstyle \sum_{k=1}^n  Z_{k}W_{k} \right |  ^2  = \left  (\sum_{k=1}^n \left| Z_{k} \right|^2  \right) \left(\sum_{k=1}^n \left| W_{k} \right|^2  \right) -\sum_{1<j<k\leq n} \left|Z_{j}\overline{W}_{k}-Z_{k}\overline{W}_{j}\right|^2. \qquad (I)\\
\end{align}
</math></center>
'''Se conoce como igualdad de Lagrange'''
'''Solución.'''
Esta demostración se hará por inducción, es decir, empezaremos suponiendo que el elemento  <math> n-1</math>  se encuentra en el conjunto, pues entonces el resultado implica que el elemento  <math>n</math>    esta en el conjunto.
Sea  <math>\eth=\left\{ \omega\in\mathbb{N\,}tal\, que\left(I\right)\, se\, cumpla\right\}
</math>
Supongamos que  <math>n-1</math>  esta en  <math>\eth</math>  , es decir,
<center><math>\begin{align}   
\left|  \textstyle \sum_{k=1}^ {n-1}  Z_{k}W_{k}  \right|=  \left(\sum_{k=1}^ {n-1}  \left|Z_{k}  \right| ^{2}\right)  \left  (\sum_{l=1}^ {n-1}  \left|W_{l}\right|^{2}\right)-\sum_{1<j<k<n-1}  \left|Z_{j}\overline{W}_{k}-Z_{k}\overline{W}_{j}\right|^{2}
    \qquad (i)\\
\end{align}
</math></center>
Tenemos que:
<center><math>\begin{align}   
\aleph=\left(\sum_{k=1}^n \left| Z_{k}\right| ^2\right) \left(\sum_{l=1}^n \left| W_{l}\right| ^2 \right)-\sum_{1<j<k\leq n} \left| Z_{j}\overline{W}_{k}-Z_{k}\overline{W}_{j}\right|^2
  \qquad (ii)\\
\end{align}
</math></center>
<center><math>\begin{align}   
\aleph= \left(\sum_{k=1}^{n-1} \left| Z_{k}\right|^2 + \left|Z_{n}\right|^2 \right) \left(\sum_{l=1}^{n-1} \left| W_{l}\right|^2+ \left|W_{n}\right|^2 \right)-\sum_{1<j<k\leq n} \left|Z_{j}\overline{W}_{k}-Z_{k}\overline{W}_{j}\right|^2
\qquad (iii)\\
\end{align}
</math></center>
<center><math>\begin{align}   
\aleph= \left(\sum_{k=1}^{n-1} \left|Z_{k}\right|^2 \right) \left(\sum_{l=1}^{n-1} \left|W_{l}\right|^2 \right)+\left|W_{n}\right|^2 \sum_{k=1}^{n-1} \left|Z_{k}\right|^2 +\left|Z_{n}\right|^2 \sum_{l=1}^{n-1} \left|W_{l}\right|^2+ \left|Z_{n}\right|^2 \left|W_{n}\right|^2 -\sum_{1<j<k\leq n}\left|Z_{j}\overline{W}_{k}-Z_{k}\overline{W}_{j}\right|^2
\qquad (iv)\\
\end{align}
</math></center>
<center><math>\begin{align} 
\aleph= \left(\sum_{k=1}^{n-1} \left|Z_{k}\right|^2 \right)\left(\sum_{l=1}^{n-1}\left|W_{l}\right|^2 \right)-\sum_{1<j<k\leq n}\left|Z_{j}\overline{W}_{k}-Z_{k}\overline{W}_{j}\right|^2 + \left|W_{n}\right|^2 \sum_{k=1}^{n-1}\left|Z_{k}\right|^2+\left|Z_{n}\right|^2 \sum_{l=1}^{n-1}\left|W_{l}\right|^2 + \left|Z_{n}\right|^2 \left|W_{n}\right|^2
\qquad (v)\\
\end{align}
</math></center>
Pero veamos la forma que toman las siguientes expresiones al expandir la suma,
<center><math>\begin{align} 
\sum_{1<j<k\leq n} \left|Z_{j}\overline{W}_{k}-Z_{k}\overline{W}_{j}\right|^2 = \left|Z_{1}\overline{W}_{2}-Z_{2}\overline{W}_{1}\right|^2 +\left|Z_{1}\overline{W}_{3}-Z_{3}\overline{W}_{1}\right|^2 +..............+\left|Z_{1}\overline{W}_{n}-Z_{1}\overline{W}_{n}\right|^2 +...........+ \left|Z_{2}\overline{W}_{3}-Z_{3}\overline{W}_{2}\right|^2 +...........+ \left|Z_{2}\overline{W}_{n}-Z_{n}\overline{W}_{2}\right|^2+..........+ \left|Z_{n-1}\overline{W}_{n}-Z_{n}\overline{W}_{n-1}\right|^2
\qquad (vi)\\
\end{align}
</math></center>
<center><math>\begin{align}
\sum_{1<j<k\leq n-1} \left|Z_{j}\overline{W}_{k}-Z_{k}\overline{W}_{j}\right|^2 = \left|Z_{1}\overline{W}_{2}-Z_{2}\overline{W}_{1}\right|^2 + \left|Z_{1}\overline{W}_{3}-Z_{3}\overline{W}_{1}\right|^2+..........+\left|Z_{1}\overline{W}_{n-1}-Z_{n-1}\overline{W}_{1}\right|^2 +..........+ \left|Z_{2}\overline{W}_{3}-Z_{3}\overline{W}_{2}\right|^2+...........+\left|Z_{2}\overline{W}_{n-1}-Z_{n-1}\overline{W}_{2}\right|^2 +...........+ \left|Z_{n-2}\overline{W}_{n-1}-Z_{n-1}\overline{W}_{n-2}\right|^2
\qquad (vii)\\
\end{align}
</math></center>
Al comparar las expresiones  <math> vi</math>  con  <math>vii</math>  se observa que:
<center><math>\begin{align}
\sum_{1<j<k\leq n} \left|Z_{j}\overline{W}_{k}-Z_{k}\overline{W}_{j} \right|^2= \sum_{1<j<k\leq n-1} \left|Z_{j}\overline{W}_{k}-Z_{k}\overline{W}_{j}\right|^2 + \sum_{j=1}^{n-1} \left|Z_{j}\overline{W}_{n}-Z_{n}\overline{W}_{j}\right|^2
\qquad (viii)\\
\end{align}
</math></center>
Entonces si ahora utilizamos las expresiones    <math>viii</math>,    <math> I</math> e  <math>i</math>  podemos re-escribir  <math>\aleph</math> de la manera siguiente:
<center><math>\begin{align}
\aleph=\left|\sum_{k=1}^{n-1}Z_{k}W_{k}\right|^2-\sum_{j=1}^{n-1}\left|Z_{j}\overline{W}_{n}-Z_{n}\overline{W}_{j}\right|^2 + \left|W_{n}\right|^2 \sum_{k=1}^{n-1}\left|Z_{k}\right|^2 + \left|Z_{n}\right|^2 \sum_{l=1}^{n-1} \left|W_{l}\right|^2 + \left|Z_{n}\right|^2 \left|W_{n}\right|^2
\qquad (ix)\\
\end{align}
</math></center>
<center><math>\begin{align}
\aleph= \left(\sum_{k=1}^n  Z_{k}W_{k} - Z_{n}W_{n}\right) \left(\sum_{l=1}^n \overline{Z}_{k}\overline{W}_{k} -\overline{Z}_{n} \overline{W}_{n}\right) - \sum_{j=1}^{n-1} \left(Z_{j}\overline{W}_{n} - Z_{n}\overline{W}_{j}\right) \left(\overline{Z}_{j}W_{n} - \overline{Z}_{n}W_{j}\right) + \left|W_{n}\right|^2 \sum_{k=1}^{n-1} \left|Z_{k}\right|^2 +\left|Z_{n} \right|^2\sum_{l=1}^{n-1}\left|W_{l}\right|^2 + \left|Z_{n}\right|^2 \left|W_{n}\right|^2
\qquad (x)\\
\end{align}
</math></center>
<center><math>\begin{align}
\aleph=\left|\sum_{k=1}^{n}Z_{k}W_{k}\right|^2 - \overline{Z}_{n}\overline{W}_{n} \sum_{k=1}^n Z_{k}W_{k}-Z_{n}W_{n} \sum_{l=1}^n \overline{Z}_{k}\overline{W}_{k}+ \left|Z_{n}W_{n}\right|^2 - \left[\sum_{j=1}^{n-1} Z_{j}\overline{W}_{n}\overline{Z}_{j}W_{n}- \sum_{j=1}^{n-1} Z_{j}\overline{W}_{n}\overline{Z}_{n}W_{j}- \sum_{j=1}^{n-1} Z_{n}\overline{W}_{j}\overline{Z}_{j}W_{n} + \sum_{j=1}^{n-1} Z_{n}\overline{W}_{j} \overline{Z}_{n}W_{j}\right] + \left|W_{n}\right|^2 \sum_{k=1}^{n-1} \left|Z_{k}\right|^2 + \left|Z_{n}\right|^2 \sum_{l=1}^{n-1} \left|W_{l} \right|^2 + \left|Z_{n}\right|^2 \left|W_{n}\right|^2
\qquad (xi)\\
\end{align}
</math></center>
<center><math>\begin{align}
\aleph=\left|\sum_{k=1}^n Z_{k}W_{k}\right|^2 - \overline{Z}_{n}\overline{W}_{n} \sum_{k=1}^n Z_{k}W_{k}-Z_{n}W_{n} \sum_{k=1}^n \overline{Z}_{k} \overline{W}_{k}+\left|Z_{n}W_{n}\right|^2 - W_{n}\overline{W}_{n} \sum_{j=1}^{n-1} Z_{j}\overline{Z}_{j}+\overline{W}_{n}\overline{Z}_{n} \sum_{j=1}^{n-1} Z_{j}W_{j}+W_{n}Z_{n} \sum_{j=1}^{n-1}\overline{W}_{j}\overline{Z}_{j}-Z_{n}\overline{Z}_{n} \sum_{j=1}^{n-1} W_{j}\overline{W}_{j} + \left|W_{n}\right|^2 \sum_{k=1}^{n-1} \left|Z_{k}\right|^2 + \left|Z_{n}\right|^2 \sum_{l=1}^{n-1} \left|W_{l}\right|^2 + \left|Z_{n}\right|^2 \left|W_{n}\right|^2
\qquad (xii)\\
\end{align}
</math></center>
<center><math>\begin{align}
\aleph=\left|\sum_{k=1}^n Z_{k}W_{k}\right|^2 - \overline{Z}_{n}\overline{W}_{n} \sum_{k=1}^n Z_{k}W_{k}-Z_{n}W_{n} \sum_{k=1}^n \overline{Z}_{k}\overline{W}_{k}+\left|Z_{n}W_{n}\right|^2- \left|W_{n}\right|^2 \sum_{j=1}^{n-1} \left|Z_{j}\right|^2 + \overline{W}_{n}\overline{Z}_{n} \sum_{j=1}^{n-1} Z_{j}W_{j}+W_{n}Z_{n} \sum_{j=1}^{n-1} \overline{W}_{j}\overline{Z}_{j}-\left|Z_{n}\right|^2 \sum_{j=1}^{n-1} \left|W_{j}\right|^2+\left|W_{n}\right|^2 \sum_{k=1}^{n-1} \left|Z_{k}\right|^2 + \left|Z_{n}\right|^2 \sum_{l=1}^{n-1} \left|W_{l}\right|^2 + \left|Z_{n}\right|^2 \left|W_{n}\right|^2
\qquad (xiii)\\
\end{align}
</math></center>
<center><math>\begin{align}
\aleph=\left| \sum_{k=1}^n Z_{k}W_{k}\right|^2 - \overline{Z}_{n} \overline{W}_{n} \sum_{k=1}^n Z_{k}W_{k}-Z_{n}W_{n} \sum_{k=1}^n \overline{Z}_{k}\overline{W}_{k}+\left|Z_{n}W_{n}\right|^2+ \overline{W}_{n}\overline{Z}_{n} \sum_{j=1}^{n-1}Z_{j}W_{j}+W_{n}Z_{n} \sum_{j=1}^{n-1} \overline{W}_{j}\overline{Z}_{j}+\left|Z_{n}W_{n}\right|^2
\qquad (xiv)\\
\end{align}
</math></center>
<center><math>\begin{align}
\aleph=\left|\sum_{k=1}^n Z_{k}W_{k}\right|^2 - \overline{Z}_{n} \overline{W}_{n}Z_{n}W_{n}-Z_{n}W_{n} \overline{Z}_{n}\overline{W}_{n}+\left|Z_{n}W_{n}\right|^2+\left|Z_{n}W_{n}\right|^2
\qquad (xv)\\
\end{align}
</math></center>
<center><math>\begin{align}
\aleph=\left|\sum_{k=1}^n Z_{k}W_{k}\right|^2
\qquad (xvi)\\
\end{align}
</math></center>
por lo tanto si  <math>n-1</math>  <math>\in</math>    <math>\eth</math>    <math>\Longrightarrow</math>      <math>n\in</math>    <math>\eth</math> .
--[[Usuario:Dali|Dali]] 03:31 14 oct 2009 (UTC)
'''2.- Sean <math>z_{1},z_{2},...,z_{n}\,</math> numeros complejos, ¿Bajo que condiciones se tiene que <math>|z_{1}+z_{2}+...+z_{n}| = |z_{1}|+|z_{2}|+...+|z_{n}|\,</math> ?'''
Si <math>z_{1} = z_{2} = ... = z_{n}\,</math>, entonces
<math>|z_{1}|+|z_{2}|+...+|z_{n}| = n|z_{1}|\,</math>
por otro lado
<math>|z_{1}+z_{2}+...+z_{n}| = |nz_{1}| = n|z_{1}|\,</math>
y por lo tanto
<math>|z_{1}|+|z_{2}|+...+|z_{n}| = |z_{1}+z_{2}+...+z_{n}|\,</math>
--[[Usuario:Luis Nava|Luis Nava]] 02:52 5 oct 2009 (UTC)
'''2.- Sean <math>z_{1},z_{2},...,z_{n}\,</math> numeros complejos, ¿Bajo que condiciones se tiene que <math>|z_{1}+z_{2}+...+z_{n}| = |z_{1}|+|z_{2}|+...+|z_{n}|\,</math> ?'''
Tomando como punto de partida la demostracion de la desigualdad del triangulo. Podemos generalizar por medio de la inducción matemática a sumas de cualquier número finito de términos.
<math>\left|z_{1}+z_{2}+...+z_{n}\right|\leq\left|z_{1}\right|+\left|z_{2}\right|+...+\left|z_{n}\right|,(n = 1,2,3,...)\qquad (1)</math>.
Es claro ver que cuando n = 2 se cumple la desigualdad del triangulo.
<math>\left|z_{1}+z_{2}\right|\leq\left|z_{1}\right|+\left|z_{2}\right|</math>
Ahora suponiendo que (1) es válida cuando n=m, debe ser tambien para n=m+1, y aplicando la desigualdad del triangulo:
<math>\left|\left(z_{1}+z_{2}+...+z_{m}\right)+z_{m+1}\right|\leq\left|z_{1}+z_{2}+...+z_{m}\right|+\left|z_{m+1}\right|</math>
<math>\left|(z_{1}+z_{2}+...+z_{m})+z_{m+1}\right|\leq(\left|z_{1}\right|+\left|z_{2}\right|+...+\left|z_{m}\right|)+\left|z_{m+1}\right|</math>
En donde los primero m terminos los redefiniremos como Z<math>_{I},</math> y el termino m+1 como Z<math>_{II},</math>. Entonces lo anterior queda como:
<math>\left|Z_{I}+Z_{II}\right|\leq\left|Z_{I}\right|+\left|Z_{II}\right|</math>
Ahora escribiendolo como igualdad
<math>\left|Z_{I}+Z_{II}\right|=\left|Z_{I}\right|+\left|Z_{II}\right|\equiv\left|z_{1}+z_{2}+...+z_{m}+z_{m+1}\right|=\left|z_{1}\right|+\left|z_{2}\right|+...+\left|z_{m}\right|+\left|z_{m+1}\right|</math>
<math>\left|z_{1}+z_{2}+...+z_{m}+z_{m+1}\right|=\left|z_{1}\right|+\left|z_{2}\right|+...+\left|z_{m}\right|+\left|z_{m+1}\right|\equiv\left|z_{1}+z_{2}+...+z_{n}\right|=\left|z_{1}\right|+\left|z_{2}\right|+...+\left|z_{n}\right|</math>
Esta desigualdad solo se cumple como igualdad cuando los numeros son colineales, ya sea en el eje real (Re) o en el imaginario (Im).
Esto es muy claro ver cuando n=2, en una grafica.
--[[Usuario:Oscar Adrian|Oscar Adrian]] 03:07 6 oct 2009 (UTC)


----
----
'''3. Encuentre el ínfimo de <math>\left | z^3 + 2 i \right |</math> en la región <math>\left \{ z \mid |z| \ge 2 \right \}</math>, y describa en qué puntos se alcanza.'''
Contribución de:[[Usuario:Karla|Karla]] 22:47 4 oct 2009 (UTC)Sanchez
 
 
Con una variante de la desigualdad del triángulo, tenemos que
 
<center><math>\left | z^3 + i \right | \ge \left | |z|^3 -1 \right | \ge 8-1. </math></center>
Por tanto,
<center><math> 7 \le \left | z^3 + i \right |. \qquad (1) </math></center>
 
Entonces, el ínfimo de la expresión es 7.
 
Por otro lado, tenemos que, si <math>z = r \left ( cos \theta + 1 \sin \theta \right )</math>
 
<center><math>
\begin{align}
\left | z^3 + i \right | ^2 & = \left | r ^3 \left ( \cos 3\theta + i \sin 3\theta \right ) + 1 \right | ^2 \\
                            & = \left | r^3 \cos 3\theta + 1 + i r^3 \sin 3\theta \right | ^2 \\
                            & = r^6 + 2 r^3 \cos 3\theta + 1. \\
\end{align}
</math></center>
 
Si tomamos la cota inferior, <math>\left | z \right | = 2</math>, la expresión anterior es entonces:
 
<center><math>
\begin{align}
\left | z^3 + i \right | ^2 & = r^6 + 2 r^3 \cos 3\theta + 1 \\
                            & = 65 + 2 r^3 \cos 3\theta. \\
\end{align}
</math></center>
 
Ya que la función coseno tiene su mínimo en el valor -1, tomemos una <math>\theta</math> tal que <math>\cos 3\theta {{=}} -1</math>. Para este caso, tenemos dos valores: <math>\theta_1 {{=}} \frac {\pi}{3}</math> y <math>\theta_2 {{=}} \pi </math>,
 
de tal forma que, con estos valores,
<center><math>\left | z^3 + i \right | ^2 = 65 - 16 = 49. </math></center>
 
Con la fórmula de De Moivre, tenemos que el ínfimo de la expresión dada toma ese valor en <math>z_1</math> y <math>z_2</math> tales que
 
<center><math>
\begin{align}
z_1 & = 2 \left ( \cos \theta_1 + i \sin \theta_1 \right ) \\
    & = 2 \left ( \cos \frac {\pi}{3}  + i \sin \frac {\pi}{3} \right )\\
    & = 2 \left ( \frac{1}{2} + i \frac{\sqrt{3}}{2} \right ) \\
z_1 & = 1 + i \sqrt{3}, \qquad (2)\\
\end{align}
</math></center>
 
y
 
<center><math>
\begin{align}
z_2 & = 2 \left ( \cos \theta_2 + i \sin \theta_2 \right ) \\
    & = 2 \left ( \cos \pi  + i \sin \pi \right )\\
z_2 & = -2. \qquad (3)\\
\end{align}
</math></center>
 
Pero, además, por le geometría de los números complejos, tenemos otros dos valores <math>z_3</math> y <math>z_4</math> tales que
 
<center><math>
\begin{align}
z_3 & = 2 \left ( \cos \theta_1+\pi + i \sin \theta_1+\pi \right ) \\
    & = 2 \left ( \cos \frac {4\pi}{3}  + i \sin \frac {4\pi}{3} \right )\\
    & = 2 \left ( - \frac{1}{2} - i \frac{\sqrt{3}}{2} \right ) \\
    & = -1 - i \sqrt{3} \\
z_3 & = - z_1, \qquad (4)\\
\end{align}
</math></center>
 
y
 
<center><math>
\begin{align}
z_4 & = 2 \left ( \cos \theta_2+\pi + i \sin \theta_2+\pi \right ) \\
    & = 2 \left ( \cos 2\pi  + i \sin 2\pi \right )\\
    & = 2 \\
z_4 & = -z_2. \qquad (5)\\
\end{align}
</math></center>
 
Por tanto, las expresiones (2), (3), (4) y (5) nos proporcionan los valores en que el ínfimo es tomado, a saber, <math>\pm (1 + i \sqrt{3})</math> y <math>\pm 2</math>.
 
--[[Usuario:Belen|Belen]] 04:08 12 oct 2009 (UTC)
 
----
----


[[Compleja:ej-cap1.1]]
[[Compleja:ej-cap1.2]]
[[Compleja:ej-cap1.2]]
[[Compleja:ej-cap1.3]]
[[Compleja:ej-cap1.4]]


[[Compleja:ej-cap2.1]]
[[Compleja:ej-cap2.2]]
[[Compleja:ej-cap2.3]]
[[Compleja:ej-cap2.4]]
[[Compleja:ej-cap2.5]]


 
[[Compleja:ej-cap3.1]]
 
[[Compleja:ej-cap3.2]]
 
[[Compleja:ej-cap3.3]]
 
[[Compleja:ej-cap3.4]]
 
 
 
 
 
 
 
 
 
 
 
 
 
 
 


[[categoría:Compleja]]
[[categoría:Compleja]]
[[categoría:Cursos]]

Revisión actual - 04:45 3 nov 2023

1. Demuestre que el producto de números complejos cumple con la ley asociativa

Sean con


Por demostrar




Por otra parte



Entonces se cumple .



Contribución de: Gabita 22:15 28 sep 2009 (UTC)



SECCIÓN 1.1.2

1. Demuestre que

Sean





Por otra parte



Contribución de: Gabita 22:15 28 sep 2009 (UTC)


2. Exprese de la forma


Por las propiedades ,



Simplificando, se obtiene:



Resolviendo la división de números complejos, de la forma:


:



=.



Contribución de:Josua Da Vinci 23:00 28 sep 2009 (UTC)



3. Demuestre que es raiz de un polinomio real si y solo si lo es.


Sea solucion de un polinomio real,

entonces

como , por lo tanto tambien es solucion.



Contribución de: Luis Nava 06:35 30 sep 2009 (UTC)


5. Sean tales que cumplen $\frac{z_2 - z_1}{z_3 - z_1} = \frac{z_1 - z_3}{z_2 - z_3}$, demuestre que estos tres puntos determinan un triángulo equilátero.


Figura 1

Tenemos que

$

y, por lo tanto,

De la Figura 1, vemos que cada una de esas normas de números complejos son exactamente los segmentos de recta que constituyen el triángulo ABC, a saber:

De (2) y (3) tenemos que:

Por triángulos semejantes, se tiene que el ángulo es igual al ángulo y éste a su vez al ángulo , es decir,

Y la ecuación (5) es precisamente la condición para que el triángulo ABC de la Figura 1 sea equilátero.


Contribución de:Belen 02:48 29 sep 2009 (UTC)


6. Sea , pruebe que


Puesto que el número complejo z puede escribirse como


Se deduce que


Como el cuadrado de un número real no puede ser negativo


Entonces

O sea

O de otra manera


Sumando , a ambos lados se tiene


Como

Entonces

De donde

Sacando raíces cuadradas positivas


Por lo tanto


Contribución de: Ralf Gutierrez 19:18 29 sep 2009 (UTC)


6-bis. Sea , pruebe que

Tenemos que , entonces de la teoria sabemos que



Tambien es inmediato que para z , , y que el cuadrado de cualquier numero real es siempre positivo, entonces de esto se tiene que

Desarrollando el binomio se tiene que


Y por la identidad (1) esto se puede escribir como

Ahora sumando en ambos lados obtenemos lo siguiente



Pero ademas como , lo sustituimos en el resultado anterior



Es facil ver que



Utilizando este resultado se deduce que



Y tomando las raices positivas llegamos al siguiente resultado



Que es lo que se queria mostrar.



Contribución de: Oscar Adrian 03:56 1 oct 2009 (UTC)


REVISADO

7. Demuestre que en un paralelogramo la suma de los cuadrados de las diagonales es la suma de los cuadrados de los lados.

Complejaej-cap1.1Paralelogramo.svg

Sacamos las normas de los números complejos

|z|= |w|=

Por algebra de vectores

Donde |h| es la resultante de |z|+|w|

+

De la misma forma el dibujo nos indica q trasladamos la magnitudes de los vectores

|w| y |z| y por tanto también tendremos |z|+|w| =|h|

Entonces si |z|+|w| = |h|

Aplicando el teorema de pitagoras q nos dice

d = cateto

f = cateto

entonces tenemos que

Aplicamos pitagoras

Por tanto

se cumple la suma de los cuadrados de los lados es igual a la suma de los cuadrados de la diagonal.


Contribución de:Karla 22:47 4 oct 2009 (UTC)Sanchez


Compleja:ej-cap1.1 Compleja:ej-cap1.2 Compleja:ej-cap1.3 Compleja:ej-cap1.4

Compleja:ej-cap2.1 Compleja:ej-cap2.2 Compleja:ej-cap2.3 Compleja:ej-cap2.4 Compleja:ej-cap2.5

Compleja:ej-cap3.1 Compleja:ej-cap3.2 Compleja:ej-cap3.3 Compleja:ej-cap3.4